2014 dxdy logo

Научный форум dxdy

Математика, Физика, Computer Science, Machine Learning, LaTeX, Механика и Техника, Химия,
Биология и Медицина, Экономика и Финансовая Математика, Гуманитарные науки


Правила форума


Посмотреть правила форума



Начать новую тему Ответить на тему
 
 Ещё комплан
Сообщение18.03.2018, 17:13 


08/12/17
255
Пусть $f(z)$ голоморфна в круге $D$ с центром в начале координат и радиусом $R$, непрерывна в замыкании этого круга. $M=\max\limits_{\left\lvert z\right\rvert=R}\left\lvert f(z)\right\rvert$.
Необходимо доказать, что при $n\in \mathbb{N}\cup 0$ и $\left\lvert z\right\rvert<R$ справедливы:
1) $\left\lvert \frac{f^{(n)}(z)}{n!}\right\rvert\leqslant \frac{MR}{(R-\left\lvert z\right\rvert)^{n+1}}$
2) $\left\lvert f'(z)\right\rvert\leqslant \frac{MR}{R^2-\left\lvert z\right\rvert^2}$

Из своих мыслей лишь оценка $\left\lvert \int\limits_{\gamma}^{} f(z)d\gamma\right\rvert\leqslant M\left\lvert\gamma\right\rvert$, где $\gamma = R e^{i\varphi}$

 Профиль  
                  
 
 Re: Ещё комплан
Сообщение18.03.2018, 17:17 
Заслуженный участник
Аватара пользователя


27/12/17
1412
Антарктика
Тут надо представить $f(z)$ в виде интеграла типа Коши и вспомнить формулу его $n$-й производной

 Профиль  
                  
 
 Re: Ещё комплан
Сообщение18.03.2018, 22:32 


08/12/17
255
Обозначим $\gamma_1=R e^{it}$ окружность с центром в нуле и радиусом R,
$\gamma_2=z e^{it}$ окружность с центром в нуле и радиусом z.
$G$ - область, ограниченная этими окружностями.
Тогда $\left\lvert \frac{f^{n}(z)}{n!}\right\rvert=\left\lvert \frac{1}{2\pi i}\int\limits_{\partial G}^{}\frac{f(\zeta)d\zeta}{(\zeta-z)^{n+1}}\right\rvert=\frac{1}{2\pi}\left\lvert \int\limits_{\gamma_1}^{}\frac{f(\zeta)d\zeta}{(\zeta-z)^{n+1}}-\int\limits_{\gamma_2}^{}\frac{f(\zeta)d\zeta}{(\zeta-z)^{n+1}}\right\rvert\leqslant\frac{1}{2\pi}\left\lvert \int\limits_{\gamma_1}^{}\frac{f(\zeta)d\zeta}{(\zeta-z)^{n+1}}\right\rvert\leqslant \frac{1}{2\pi}\int\limits_{\gamma_1}^{}\left\lvert \frac{f(\zeta)}{(\zeta-z)^{n+1}}\right\rvert \left\lvert d\gamma_1\right\rvert\leqslant \frac{1}{2\pi}\frac{M}{(R-\left\lvert z\right\rvert)^{n+1}}\int\limits_{0}^{2\pi}Rdt=\frac{1}{2\pi}\frac{M}{(R-\left\lvert z\right\rvert)^{n+1}}2\pi R=\frac{MR}{(R-\left\lvert z\right\rvert)^{n+1}}$
Истина? Или где-то рядом? Или где-то там?

 Профиль  
                  
 
 Re: Ещё комплан
Сообщение19.03.2018, 03:19 
Заслуженный участник
Аватара пользователя


27/12/17
1412
Антарктика
Второй интеграл вообще ни к чему, можно сразу брать по большой окружности, т.к. интеграл типа Коши определен на любой кривой, лишь бы через точку $z$ не проходила

-- 19.03.2018, 05:22 --

А вот во втором примере, похоже, так просто не выйдет. Придется более аккуратно оценивать, т.е. сначала параметризовать интеграл, преобразовывать, а модули навешивать в последнюю очередь. По-крайней мере мне так кажется на первый взгляд.

 Профиль  
                  
 
 Re: Ещё комплан
Сообщение19.03.2018, 15:38 
Заслуженный участник
Аватара пользователя


27/12/17
1412
Антарктика
Подсказка по второй задаче (т.к. она далеко не такая простая, как первая, и я сам не сразу допёр): $\left\lvert\xi-z\right\rvert^2=(\xi-z)(\bar{\xi}-\bar{z})$, затем замена $\xi=Re^{i\varphi}$.
В итоге получается интеграл $\int\limits_{0}^{2\pi}\frac{1}{R^2+\left\lvert{z}\right\rvert^2-2R\left\lvert{z}\right\rvert\cos{\psi}}d\psi$ (буква $\psi$ написана не случайно), ну и затем этот интеграл надо посчитать. Вот его значение и даст нужный ответ. Просто оценками тут не отделаться.

 Профиль  
                  
 
 Re: Ещё комплан
Сообщение19.03.2018, 19:53 


08/12/17
255
$\psi=\varphi+\alpha$, где $\alpha=\arg \frac{\overline{z}}{\left\lvert z\right\rvert}$. Верно расшифровал?

 Профиль  
                  
 
 Re: Ещё комплан
Сообщение20.03.2018, 03:19 
Заслуженный участник
Аватара пользователя


27/12/17
1412
Антарктика
Ага (правда, там косинус разности). Еще, лучше интеграл считать в пределах $[-\pi,\pi]$

 Профиль  
                  
 
 Re: Ещё комплан
Сообщение20.03.2018, 09:39 


08/12/17
255
thething в сообщении #1298457 писал(а):
пределах $[-\pi,\pi]$

У меня в итоге получается так
$\frac{4}{R^2-\left\lvert z\right\rvert^2}\arctg \frac{(R+\left\lvert z\right\rvert)\tg\frac{\psi}{2}}{R-\left\lvert z\right\rvert}$ от 0 до $\pi$, что равно $\frac{4}{R^2-\left\lvert z\right\rvert^2}(\arctg(\infty)-\arctg 0)=\frac{2\pi}{R^2-\left\lvert z\right\rvert^2}$. Насколько верно? Или как-то лучше и проще можно?

 Профиль  
                  
 
 Re: Ещё комплан
Сообщение20.03.2018, 11:04 
Заслуженный участник
Аватара пользователя


27/12/17
1412
Антарктика
Ответ-то в-общем правильный, но почему такой ответ только на половине отрезка, когда должен получиться на всем? Или Вы четность учли? Если да, то все верно. Если нет, то в каком-то месте в два раза ошиблись.

Касаемо проще -- никак особо проще не будет. Универсальная тригонометрическая подстановка, которую Вы использовали, самый простой путь.

 Профиль  
                  
 
 Re: Ещё комплан
Сообщение20.03.2018, 15:28 


08/12/17
255
thething в сообщении #1298477 писал(а):
Или Вы четность учли?

Да, учёл чётность. Поэтому удвоил интеграл. $2\pi$ - итоговый ответ.

 Профиль  
                  
 
 Re: Ещё комплан
Сообщение20.03.2018, 15:33 
Заслуженный участник
Аватара пользователя


27/12/17
1412
Антарктика
Тогда все правильно

 Профиль  
                  
Показать сообщения за:  Поле сортировки  
Начать новую тему Ответить на тему  [ Сообщений: 11 ] 

Модераторы: Модераторы Математики, Супермодераторы



Кто сейчас на конференции

Сейчас этот форум просматривают: dgwuqtj, mihaild


Вы не можете начинать темы
Вы не можете отвечать на сообщения
Вы не можете редактировать свои сообщения
Вы не можете удалять свои сообщения
Вы не можете добавлять вложения

Найти:
Powered by phpBB © 2000, 2002, 2005, 2007 phpBB Group